Proving a/b < (a+c)/(b+d) for Positive Numbers: A First Course in Calculus

  • Thread starter Thread starter dmcharg
  • Start date Start date
Click For Summary
The discussion centers on proving the inequality a/b < (a+c)/(b+d) given that a/b > c/d for positive numbers a, b, c, and d. The initial approach suggested multiplying both sides of the inequality by b and b+d, but it was pointed out that this leads to the incorrect conclusion. Instead, the correct manipulation shows that if a/b > c/d, then the inequality actually reverses, leading to a/b > (a+c)/(b+d). The importance of maintaining the direction of the inequality when multiplying by positive values is emphasized. This highlights the need for careful handling of inequalities in calculus proofs.
dmcharg
Messages
6
Reaction score
0
Hi
I am working my way through Serge Langs A first course in Calculus and have encountered this question/proof which i am not sure how to do. Any assistance much appreciated.

Let a,b,c,d > 0 such that a/b > c/d Prove that

a/b < (a+c)/(b+d)

?

Thanks
David.
 
Last edited:
Physics news on Phys.org
IN which section is this? The surrounding material may well be helpful - is it near stuff on cauchy schwartz, or the triangle inequality? Or something else entirely? My experience with this type of question is that it is 'easy' with the right method, and impossible if you don't know/guess it. Any similar questions in the text near this one may well give you plenty of insight.
 
My first thought would be to multiply both sides of the inequality by b and b+ d.
 
dmcharg said:
Let a,b,c,d > 0 such that a/b > c/d Prove that

a/b < (a+c)/(b+d)

It is wrong.
If a/b > c/d , then
ad > bc , or
ab+ad > ab+bc , or
a(b+d) > b(a+c) , or
a/b > (a+c)/(b+d)

That's all.
 
Thanks. Yes i got the direction of the inequality the wrong way round but i see your approach. First multiply both side by b, then d and then add ab to both sides, all of which can only be done on the assumption that all values are > 0 and hence preserving the direction of the inequality.
 
Question: A clock's minute hand has length 4 and its hour hand has length 3. What is the distance between the tips at the moment when it is increasing most rapidly?(Putnam Exam Question) Answer: Making assumption that both the hands moves at constant angular velocities, the answer is ## \sqrt{7} .## But don't you think this assumption is somewhat doubtful and wrong?

Similar threads

Replies
2
Views
2K
  • · Replies 6 ·
Replies
6
Views
1K
  • · Replies 7 ·
Replies
7
Views
2K
Replies
3
Views
2K
  • · Replies 9 ·
Replies
9
Views
2K
  • · Replies 5 ·
Replies
5
Views
1K
  • · Replies 26 ·
Replies
26
Views
3K
  • · Replies 16 ·
Replies
16
Views
2K
  • · Replies 2 ·
Replies
2
Views
1K
Replies
7
Views
1K